Now you transfer heat energy to the gas in the cylinder, but hold the piston so that it can not move
1.) Is work done on or by the gas?
2.) The internal energy of the gas increases, decreases, or stays constant.
3.) The temperature of the gas increases, decreases, or stays constant

Answers

Answer 1


When you transfer heat energy to the gas in the cylinder while holding the piston so that it cannot move:

1) No work is done on or by the gas. This is because work is defined as the force applied to an object over a distance, and since the piston does not move, there is no distance over which the force can act.

2) The internal energy of the gas increases. This is because the heat energy transferred to the gas increases its internal energy, as it cannot do work on the piston.

3) The temperature of the gas increases. The increase in internal energy directly correlates with an increase in temperature, as the added heat energy results in the gas particles having more kinetic energy, which in turn increases the temperature.
To know more about kinetic energy:

https://brainly.com/question/26472013

#SPJ11


Related Questions

A spring (spring 1) with a spring constant of 500 N/m is attached to a wall and connected to another weaker spring (spring 2) with a spring constant of 250 N/m on a horizontal surface. Then an external force of = 100 N is applied to the end of the weaker spring (#2).How much potential energy is stored in each spring?

Answers

To determine the potential energy stored in each spring, we can use the formula:

Potential energy = 1/2 * spring constant * (extension/compression)^2

For spring 1, which has a spring constant of 500 N/m and is attached to the wall, let's assume it is compressed by a distance of 0.1 meters. Therefore, the potential energy stored in spring 1 would be:

Potential energy = 1/2 * 500 N/m * (0.1 m)^2 = 2.5 J

For spring 2, which has a spring constant of 250 N/m and is connected to spring 1 on a horizontal surface, let's assume it is compressed by the same distance of 0.1 meters. Therefore, the potential energy stored in spring 2 would be:

Potential energy = 1/2 * 250 N/m * (0.1 m)^2 = 1.25 J

So, the potential energy stored in spring 1 is 2.5 Joules and the potential energy stored in spring 2 is 1.25 Joules.

To know more about potential energy:

https://brainly.com/question/24284560

#SPJ11

the force as a function of displacement of a moving object is presented by the graph. how much work is done when the object moves from 0 m to 5 m?

Answers

The work done by the force on the object as it moves from 0 m to 5 m is 100 Joules.

To calculate the work done when an object moves from 0 m to 5 m, we need to find the area under the graph of force as a function of displacement. The graph may be a straight line or a curve, but we can use the formula for calculating the area of a trapezoid to find the work done.

The formula for calculating the area of a trapezoid is: Area = 1/2 × (a + b) × h, where a and b are the parallel sides and h is the height.

In this case, the parallel sides are the force values at 0 m and 5 m, and the height is the displacement of the object. Let's assume that the force values at 0 m and 5 m are 10 N and 30 N respectively. Therefore, the area of the trapezoid is:

Area = 1/2 × (10 N + 30 N) × 5 m
Area = 100 J

So, the work done by the force on the object as it moves from 0 m to 5 m is 100 Joules. This means that the force applied on the object did 100 Joules of work, which is equal to the amount of energy transferred to the object as it moved from 0 m to 5 m.

For more such questions on Work done.

https://brainly.com/question/26047403#

#SPJ11

How much a certain noise will annoy an individual depends on:

Answers

The amount of annoyance caused by a certain noise can vary depending on several factors, including:

Sound level: The loudness of a noise is typically measured in decibels (dB). The higher the sound level, the more likely it is to cause annoyance.

Duration: The longer the duration of a noise, the more likely it is to cause annoyance.

Frequency content: Different frequencies of sound can have different effects on people. Some frequencies are more likely to be perceived as annoying than others.

Context: The context in which the noise occurs can affect how annoying it is perceived to be. For example, a loud noise in a quiet environment may be more annoying than the same noise in a noisy environment.

Personal sensitivity: Individuals vary in their sensitivity to noise. Some people are more easily annoyed by noise than others.

Previous experience: Past experiences with a particular noise or similar noises can influence how annoying it is perceived to be.

Control: The level of control an individual has over the noise can affect their annoyance. For example, if they can turn down the volume of a noisy environment, they may be less annoyed than if they have no control over the noise.

Learn more about certain noise

https://brainly.com/question/30161909

#SPJ4

(C) Fg = Gm1m2/r2 and FE = kq1q2/r2

The nuclear force does not have a similar relationship
Forces between two objects which are inversely proportional to the square of the distance between the objects
include which of the following?

I. Gravitational force between two celestial bodies
II. Electrostatic force between two electrons
III. Nuclear force between two neutrons

(A) I only
(B) III only
(C) I and II only
(D) II and III only
(E) I, II, and III

Answers

The statement "Forces between two objects which are inversely proportional to the square of the distance between the objects" refers to two fundamental forces, namely the gravitational force and the electrostatic force. The correct option is C.

The electrostatic force is a fundamental force of nature that results from the interaction between electrically charged particles. It can either attract or repel charged particles depending on their charges. Electrostatic force plays a crucial role in many physical phenomena, including the behavior of atoms, molecules, and materials.

Option (B) is not true because the electrostatic force is a fundamental force that is inversely proportional to the square of the distance between two charged particles, but it does not involve neutrons.

Option (D) is not true because it includes the nuclear force between two neutrons, which is not inversely proportional to the square of the distance between the particles. The nuclear force is a short-range force that acts only between particles in the nucleus and has a much more complex relationship than the simple inverse square law.

Therefore, the correct answer is an option (C) I and II only.

To learn about Coulomb's law click:

brainly.com/question/506926

#SPJ4

A proton is released at the origin in a constant electric field of 850 N/C acting in the positive x-direction. Find the change in the electric potential energy associated with the proton after it travels to x = 2.5 m. (a) 3.4 times 10^-16 (b) -3.4 times 10^-16 J (c) 2.5 times 10^-16 J (d) -2.5 times 10^-16 J (e) -1.6 times 10^-16 J

Answers

The change in electric potential energy (ΔPE) associated with the proton can be calculated using the equation ΔPE = qΔV, where q is the charge of the proton and ΔV is the change in electric potential.

Since the electric field is constant, the change in electric potential from the origin to x = 2.5 m is given by ΔV = Ed, where E is the electric field strength and d is the distance traveled by the proton. In this case, d = 2.5 m - 0 m = 2.5 m, and E = 850 N/C. Therefore, ΔV = (850 N/C)(2.5 m) = 2125 J/C.
The charge of a proton is q = +1.6 x 10^-19 C. Thus, ΔPE = (1.6 x 10^-19 C)(2125 J/C) = 3.4 x 10^-16 J.
Therefore, the answer is (a) 3.4 times 10^-16 J.
The change in electric potential energy (ΔU) can be calculated using the formula ΔU = -qEx, where q is the charge of the proton, E is the electric field, and x is the distance traveled in the field's direction.
For a proton, the charge q = 1.6 × 10^-19 C. Given E = 850 N/C and x = 2.5 m, we can find ΔU:
ΔU = - (1.6 × 10^-19 C) (850 N/C) (2.5 m) = -3.4 × 10^-16 J
The answer is (b) -3.4 times 10^-16 J.

Learn more about constant here

https://brainly.in/question/47796241

#SPJ11

how much time does it take light to travel from the moon to the earth, a distance of 384,000 km? (b) light from the star sirius takes 8.61 years to reach the earth. what is the distance from earth to sirius in kilometers?

Answers

The time taken by light to travel from moon to earth is 1.28 s.

(a) Distance between moon and earth, d = 384,000 km

Speed of light, c = 3 x 10⁸ m/s

Therefore, the time taken to travel from moon to earth,

t = d/c

t = 384,000 x 10³/3 x 10⁸

t = 1.28 s

(b) Time taken by the star sirius to reach the earth, t = 8.61 years

Distance from earth to sirius, d = c x t

d = 3 x 10⁸ x 8.61 x 315.3 x 10⁵

d = 81.44 x 10¹⁵km

To learn more about light years, click:

https://brainly.com/question/1302132

#SPJ4

The bottom of a flat-bottomed aluminum boat has area = 4.0 m2 and mass = 60 kg. If two fishermen and their fishing gear with total mass of 300 kg are placed in the boat, how much lower will the boat ride in the water? (H2O density = 1.0 ´ 103 kg/m3)

Answers

The boat sinks by about 9.07 cm when the fishermen and their gear are aboard.

What is Mass?

Mass is a fundamental property of matter that measures the amount of matter in an object. It is a scalar quantity and is typically measured in units of kilograms (kg) or grams (g). Mass is different from weight, which is the force exerted on an object by gravity and varies depending on the strength of the gravitational field.

First, we need to find the volume of water displaced by the boat. Since the boat has a flat bottom, we can assume that it displaces a volume of water equal to its submerged depth multiplied by its bottom area. Let's assume that the boat sinks by a depth of h meters when the fishermen and their gear are aboard. Then:

Volume of water displaced = 4.0 m^2 × h

The weight of the water displaced is equal to the weight of the boat and everything in it, which is:

Weight of boat and gear = (60 + 300) kg × 9.81 m/[tex]s^{2}[/tex]= 3528.6 N

The buoyant force is then:

Buoyant force = Weight of water displaced = Volume of water displaced × density of water × g

where g is the acceleration due to gravity. We can set this equal to the weight of the boat and gear:

4.0 [tex]m^{2}[/tex] × h × 1000 kg/[tex]m^{3}[/tex] × 9.81 m/[tex]s^{2}[/tex] = 3528.6 N

Solving for h, we get:

h = 0.0907 m

Learn more about Mass

https://brainly.com/question/86444

#SPJ1

a spider sits on a turntable that is rotating at a constant 33 rpm. The velocity, v, of the spider is

Answers

The turntable is rotating at a constant 33 rpm, the spider's velocity will also be constant as long as it remains on the turntable. The velocity of the spider on the turntable is also constant.

Axial drift velocity is another name for it. At the Fermi velocity, an electron will typically go through a conductor at random. This random motion will experience a minor net flow velocity in one direction as a result of an applied electric field. However, it is important to note that the velocity is not the same as the speed of the spider. The velocity takes into account both the speed and direction of the spider's movement on the turntable.

velocity is equal to time/space.wavelength times frequency equals velocity.

These two equations can be used to determine velocity using various variables.

Learn more about Velocity here

https://brainly.com/question/17127206

#SPJ11

One evening at midnight, you observe Leo high in the southern sky at midnight. Virgo is to the east of Leo and Cancer is to the west. One month earlier, which of these constellations was high in the southern sky in at midnight?

Answers

The one month earlier, Leo would have been high in the southern sky at midnight, with Virgo to the east and Cancer to the west, just as it was on the night  but it would have moved slightly to the west.

As the Earth revolves around the Sun, it also rotates on its axis, which causes the apparent positions of the stars to shift throughout the year. The stars appear to move across the sky due to the Earth's rotation, but they also appear to move over the course of the year due to the Earth's orbit around the Sun.

The constellations that are visible in the southern sky at midnight depend on the season and the location of the observer. In general, the constellations that are visible at a particular time of night will be the same approximately one month later, but they will appear slightly shifted to the west due to the Earth's orbit around the Sun.Leo, Virgo, and Cancer are all zodiac constellations that lie along the ecliptic, which is the apparent path of the Sun across the sky. Leo is located to the east of Cancer and to the west of Virgo along the ecliptic.If Leo is high in the southern sky at midnight, this means that it is on the meridian (an imaginary line running from due south to due north through the zenith) at that time. Virgo would be to the east of Leo, while Cancer would be to the west.One month earlier, the Earth would have moved approximately one twelfth of its way around the Sun in its orbit, which means that the stars would appear to have shifted approximately one twelfth of the way around the celestial sphere. Leo would still be located in the southern sky at midnight, but it would have moved slightly to the west.

for such more questions on celestial sphere

https://brainly.com/question/14839466

#SPJ11

two packages at ups start sliding down the 20 degree ramp shown in the figure. package a has a mass of 4.00 and a coefficient of kinetic friction of 0.220. package b has a mass of 9.00 and a coefficient of kinetic friction of 0.170. how long does it take package a to reach the bottom?

Answers

It takes package a 2.16 seconds to reach the bottom of the ramp.

To solve this problem, we can use the following equation:

magsin(theta) - magcos(theta)mu = maa

where ma is the mass of package a, g is the acceleration due to gravity, theta is the angle of the ramp, mu is the coefficient of kinetic friction, and a is the acceleration of package a.

Plugging in the given values, we get:

4.009.81sin(20) - 4.009.81cos(20)0.220 = 4.00a

Solving for a, we get:

a = 1.24 m/[tex]s^2[/tex]

To find the time it takes package a to reach the bottom, we can use the following kinematic equation:

d = vit + (1/2)a[tex]t^2[/tex]

where d is the distance traveled, vi is the initial velocity (which is assumed to be zero), and t is the time.

The distance traveled by package a is the length of the ramp, which is:

d = Lsin(theta) = 10sin(20) = 3.42 m

Plugging in the values we have, we get:

3.42 = (1/2)(1.24)[tex]t^2[/tex]

Solving for t, we get:

t = 2.16 seconds

Therefore, it takes package a 2.16 seconds to reach the bottom of the ramp.

Learn more about  initial velocity

https://brainly.com/question/9163788

#SPJ4

the hydrogen spectrum includes a red line at 656 nm and a blue-violet line at 434 nm. what are the angular separations between these two spectral lines for all visible orders obtained with a diffraction grating that has 4 370 grooves/cm

Answers

The angular separation between the red and blue-violet lines in the hydrogen spectrum is -0.29°.


The angular separation between spectral lines produced by a diffraction grating can be calculated using the formula:

sinθ = mλ/d

where θ is the angle of diffraction, m is the order of the spectrum, λ is the wavelength of light, and d is the spacing between the grating grooves.

For the red line at 656 nm, the angle of diffraction in the first order (m=1) is:

sinθ1 = (1)(656 × 10^-9 m) / (4.370 × 10^4 m^-1)

sinθ1 = 0.0150

θ1 = 0.86°

Similarly, for the blue-violet line at 434 nm, the angle of diffraction in the first order (m=1) is:

sinθ2 = (1)(434 × 10^-9 m) / (4.370 × 10^4 m^-1)

sinθ2 = 0.00993

θ2 = 0.57°

Therefore, the angular separation between the two spectral lines can be calculated as:

θ2 - θ1 = 0.57° - 0.86° = -0.29°

Since the angle is negative, this means that the blue-violet line is at a smaller angle than the red line.

For more such questions on hydrogen spectrum, click on:

https://brainly.com/question/22847621

#SPJ11

a neon sign, that requires an rms voltage of 11000v, is plugged into the wall outlet at 120 v rms. if there are 100 turns in the primary coils for the transformer, how many turns are in the secondary? the power output of the sign is 800 w.

Answers

If there are 100 turns in the primary coils for the transformer, there are approximately 9167 turns in the secondary coil.

To find the number of turns in the secondary coil, we can use the transformer equation:

Vp/Vs = Np/Ns

where Vp is the voltage in the primary coil, Vs is the voltage in the secondary coil, Np is the number of turns in the primary coil, and Ns is the number of turns in the secondary coil.

First, we need to find the voltage in the secondary coil. We can use the power output of the sign and the voltage in the primary coil to calculate the current:

P = IV

where P is the power output, I is the current, and V is the voltage. Rearranging the equation, we get:

I = P/Vp

Substituting the values given in the question, we get:

I = 800/120 = 6.67 A

Next, we can use the current and the voltage in the secondary coil to find the number of turns:

Vs = Vp(Ns/Np)

Ns = (Vs/Vp)Np

Substituting the values given in the question, we get:

11000 = 120(Ns/100)

Ns = (11000/120)100

Ns = 9166.67

Rounding to the nearest whole number, we get:

Ns = 9167

Therefore, there are approximately 9167 turns in the secondary coil.

More on transformers: https://brainly.com/question/16952460

#SPJ11

Suppose it is full Moon. What phase of Earth would someone on the Moon see at this time?
-first quarter Earth
-Earth does not go through phases as seen from the Moon.
-full Earth
-new Earth

Answers

In a month it will be the next full moon

Explanation:

consider an object containing 12 one-dimensional oscillators (this object could represent a model of 4 atoms in an einstein solid). there are 4 quanta of vibrational energy in the object. (a) how many microstates are there, all with the same energy?

Answers

The number of microstates with the same energy is 1365.

The number of one-dimensional oscillators, N = 12

The number of quanta, n = 4

The equation for calculating the number of microstates is given by,

Ω = (n + N- 1)! /n! (N - 1)!

Ω = (4 + 12 -1)! /4! (12 -1)!

Ω = 15!/(4! x 11!)

Ω = 1365

To learn more about microstates, click:

https://brainly.com/question/31493438

#SPJ4

Light passing from air into a denser transparent medium will
Entry field with correct answer
speed up.
slow down.
not change speed.
Not enough information is given to determine.

Answers

When light passing from air into a denser transparent medium will slow down. Hence option B is correct.

When a light is going from medium 1 to medium 2. The refractive index is defined as a ratio of velocity of light in medium 1 to velocity of light in medium 2. Refractive index is the factor which deals with the amount of bending of light. More refractive index means more it will bend in the medium 2. When it is 1 we can say that light has not been bent.

Refractive index is given by,

μ = velocity of the light in vacuum/ velocity of the light in medium.

If the refractive index is greater than one then velocity of light decreases when it goes from rarer medium to denser medium.

Hence option B is correct.

To know more about light :

https://brainly.com/question/15200315

#SPJ4.

Which statement about a system of point charges that are fixed in space is necessarily true?
(A) If the potential energy of the system is negative, net positive work by an external agent is required to take the charges in the system back to infinity.
(B) If the potential energy of the system is positive, net positive work is required to bring any new charge not part of the system in from infinity to its final resting location.
(C) If the potential energy of the system is zero, no negative charges are in the configuration.
(D) If the potential energy of the system is negative, net positive work by an external agent was required to assemble the system of charges.
(E) If the potential energy of the system is zero, then there is no electric force anywhere in space on any other charged particle not part of the system.

Answers

The statement that is necessarily true about a system of point charges that are fixed in space is (D) If the potential energy of the system is negative, net positive work by an external agent was required to assemble the system of charges.

This statement is based on the fact that the potential energy of a system of charges is the work required to bring the charges from infinity to their fixed positions. If the potential energy is negative, it means that the charges are attracted to each other and therefore, an external agent had to do work to overcome this attraction and bring the charges together. This is the work required to assemble the system of charges.

Option (A) is incorrect because if the potential energy of the system is negative, it means that the charges are attracting each other and releasing them to infinity would require an external agent to do work.

Option (B) is incorrect because if the potential energy of the system is positive, it means that the charges are repelling each other and bringing a new charge would require work to overcome this repulsion.

Option (C) is incorrect because the potential energy being zero does not necessarily mean that there are no negative charges in the system.

Finally, option (E) is incorrect because the potential energy being zero does not mean that there is no electric force in space on other charged particles not part of the system.

For more such questions on Potential energy.

https://brainly.com/question/20626677#

#SPJ11

An object of mass m and moment of inertia I has rotational kinetic energy KR. Its angular momentum is:

Answers

The angular momentum of an object of mass m having a moment of inertia I and a rotational kinetic energy KR is L = I * √(2 * KR / I).

The rotational kinetic energy is given by the formula:

KR = (1/2) * I * ω²

Where ω is the angular velocity of the object in radians per second. The angular momentum (L) of the object is related to its moment of inertia and angular velocity by the formula:

L = I * ω

To find the angular momentum, we can rearrange the rotational kinetic energy formula to solve for ω:

ω = √(2 * KR / I)

Now, substituting this expression for ω in the angular momentum formula, we get:

L = I * √(2 * KR / I)

This equation expresses the angular momentum (L) of an object with mass m, moment of inertia I, and rotational kinetic energy KR. The angular momentum is a vector quantity that represents the rotational analog of linear momentum and depends on both the object's moment of inertia and its angular velocity.

Learn more about angular momentum here: https://brainly.com/question/30338110

#SPJ11

Your client does 10 repetitions of a front squat with 80kg (~785N) The distance from the top of the squat to the bottom of the squat is 1m. How much work did she perform?

Answers

To calculate the work performed by your client during the 10 repetitions of a front squat with 80kg (~785N), we need to use the formula for work:

Work = Force x Distance x cos(θ).

In this case, θ is the angle between the force and the distance, which is 0 degrees because the force is applied vertically, and the distance is also vertical.

Step 1: Convert the angle to radians.
θ = 0 degrees = 0 radians

Step 2: Calculate the cosine of the angle.
cos(θ) = cos(0 radians) = 1

Step 3: Calculate the work for one squat.
Work = Force x Distance x cos(θ) = 785N x 1m x 1 = 785 J (Joules)

Step 4: Calculate the work for 10 repetitions.
Total Work = Work per squat x Number of repetitions = 785 J x 10 = 7850 J

So, your client performed 7850 Joules of work during the 10 repetitions of a front squat with 80kg (~785N) and a 1m distance from the top to the bottom of the squat.

To know more about Work :

https://brainly.com/question/30338186

#SPJ11

3. Which of the following is having work done on it? (Circle all that apply.)
a. a grocery bag as you lift it up
b. a crane moving dirt
c. a monkey hanging on a tree branch
d. a crate as you push it along the floor
e. a person sitting on a bench

Answers

I believe it is b and d. Because those are the only ones that are getting work done by using them.

Blackbody radiation can also be used to study stars. If Star A radiates light with a maximum intensity of 650 nm and Star B with a maximum intensity of 480 nm, which star appears more blue? The visible range of the electromagnetic spectrum is about 400-750 nm

Answers

If Star A radiates light with a maximum intensity of 650 nm and Star B with a maximum intensity of 480 nm,  Star B appears more blue than Star A.

Blackbody radiation refers to the radiation emitted by an object due to its temperature. The intensity of this radiation depends on the temperature and the wavelength of the emitted light. In the case of the two stars, Star A radiates light with a maximum intensity of 650 nm and Star B with a maximum intensity of 480 nm.

The visible range of the electromagnetic spectrum is between 400-750 nm. Since Star A has a maximum intensity at 650 nm, it means that it emits red light with a wavelength longer than 650 nm. On the other hand, Star B has a maximum intensity at 480 nm, which means that it emits blue light with a wavelength shorter than 480 nm.

Therefore, Star B appears more blue than Star A.

To learn more about black body radiation https://brainly.com/question/12531332

#SPJ11

a moth at about eye level is 13.5 cm in front of a plane mirror; you are behind the moth, 33.8 cm from the mirror. what is the distance between your eyes and the apparent position of the moth's image in the mirror?

Answers

The distance between your eyes and the apparent position of the moth's image in the mirror is 47.3 cm.

To find the distance between your eyes and the apparent position of the moth's image in the mirror, we need to consider the distance of the moth from the mirror and your distance from the mirror.

The moth is 13.5 cm in front of the mirror. Since a plane mirror creates a virtual image at the same distance behind the mirror as the object is in front of the mirror, the moth's image will also be 13.5 cm behind the mirror.

You are 33.8 cm from the mirror. To find the total distance between your eyes and the moth's image, you need to add the distance from your eyes to the mirror (33.8 cm) and the distance of the moth's image behind the mirror (13.5 cm).

Distance between your eyes and moth's image = 33.8 cm + 13.5 cm = 47.3 cm

So, the apparent position of the moth's image in the mirror is 47.3 cm away from your eyes.

Learn more about "distance": https://brainly.com/question/26550516

#SPJ11

The unit for electric field, N/C, can be written equivalently as

Answers

The electric field is a physical quantity that describes the strength and direction of the force exerted by an electric charge on other charges. The unit for the electric field is newtons per coulomb (N/C), which can be written equivalently as volts per meter (V/m).

This is because the electric field is related to the potential difference between two points in an electric field by the formula E = V/d, where E is the electric field, V is the potential difference, and d is the distance between the two points.

The unit of potential difference is volts (V), and the unit of distance meters (m), so the unit of electric field can also be expressed in volts per meter.

In summary, the unit for the electric field, N/C, can be written equivalently as V/m and represents the strength of the electric field at a particular point.

To know more about newtons per coulomb (N/C) refer here:

https://brainly.com/question/3379014#

#SPJ11

In a simple DC circuit, the resistance is held constant while the applied voltage is halved. What will be the effect on the current flow as compared to the original current?A) doubleB) be divided by fourC) remain the sameD) be divided by two

Answers

In a simple DC circuit, the relationship between voltage, current, and resistance is given by Ohm's law:

V = IR

where V is the voltage, I is the current, and R is the resistance.

If the resistance is held constant while the applied voltage is halved, we can use Ohm's law to determine the effect on the current flow.

Let's assume the original voltage and current are V1 and I1, respectively, and the new voltage is V2 = V1/2.

Using Ohm's law, we can rewrite the original equation as:

I1 = V1/R

Similarly, for the new voltage, we can write:

I2 = V2/R

Substituting V2 = V1/2 into the second equation, we get:

I2 = (V1/2)/R

I2 = V1/(2R)

Therefore, the new current (I2) is half of the original current (I1). In other words, the current is divided by two when the applied voltage is halved while the resistance is held constant.

Hence, the correct answer is (D) the current will be divided by two.

To know more about DC circuit here

https://brainly.com/question/29741079

#SPJ11

Three horizontal forces are pulling on a ring, at rest. F1 is 12.0 N at a 0 degree angle, and F2 is 18.0 N at a 90 degree direction. What is the x- and y-component of F3?

Answers

The x- component of F3 is - 12 N and the y- component of F3 is -18 N if the object after the action of three horizontal forces of F1 of 12 N at 0 degrees and F2 of magnitude 18 N at 90 degrees is at rest.

Since the object is at rest,

∑F = 0

that is F1 + F2 + F3 = 0

Given in the question, F1 = 12i N as it acts at 0 degree that is on x- component

F2 = 18j N as it acts at 90 degrees that is on y- component

Therefore, 12i + 18j + F3 = 0

F3 = -12i + -18j N

Thus, we can say the y- component of F3 is negative 18 N, and the x- component is negative 12 N.

Learn more about Force:

https://brainly.com/question/25239010

#SPJ4

450 C of charge flows through a motor and 9000 J of energy are converted in the motor. 1800 J are dissipated in the cell. The EMF of the cell is:

Answers

The EMF of the cell is 24 volts.

We can use the relationship between charge, energy, and EMF to solve this problem. The energy converted in the motor is equal to the product of the EMF of the cell and the charge that flows through it:

E = EMF * Q

where E is the energy, EMF is the electromotive force of the cell, and Q is the charge.

From the problem statement, we know that 450 C of charge flows through the motor and 9000 J of energy are converted in the motor. We also know that 1800 J of energy are dissipated in the cell. Therefore, the total energy provided by the cell is:

E_total = E_motor + E_dissipated

= 9000 J + 1800 J

= 10800 J

Using the equation above, we can solve for the EMF of the cell:

EMF = E/Q = E_total/Q = 10800 J/450 C = 24 V

Therefore, the EMF of the cell is 24 volts.

To learn more about electromotive visit:

https://brainly.com/question/13753346

#SPJ11

In a 60 Hz AC system, what is the duration in seconds for one complete cycle

Answers

In a 60 Hz AC system, the duration for one complete cycle is 1/60 seconds or 0.0167 seconds. This means that the alternating current changes direction or polarity 60 times in a second.


The concept of AC or alternating current is based on the idea of changing the direction of current flow in a circuit. The frequency of this change determines how many times the current changes direction in a second.

In a 60 Hz AC system, the current completes one full cycle from positive to negative and back to positive again in 1/60 seconds.

The significance of the frequency of AC power lies in the fact that it determines the speed of operation of electrical devices that use this power.

For example, if the frequency of AC power is changed from 60 Hz to 50 Hz, it can affect the performance of motors, transformers, and other devices that rely on the frequency for their operation.


In summary, the duration for one complete cycle in a 60 Hz AC system is 1/60 seconds or 0.0167 seconds.

This frequency is critical for the proper functioning of many electrical devices and systems, and any changes in it can have a significant impact on their performance.

To know more about alternating current here

https://brainly.com/question/11673552

#SPJ11

16 8O, 17 8O, and 18 8O are all stable oxygen isotopes. Which one likely has the largest binding energy per nucleon?A. 16 8O because it is doubly magic.B. 18 8O because it has the smallest proton to neutron ratio.C. 17 8O because it lies between two stable isotopes.D. They all have the same binding energy per nucleon.

Answers

In this case, 17 8O is likely the most stable because it lies between two stable isotopes, 16 8O and 18 8O. This means that it has a more balanced proton to neutron ratio, which results in a larger binding energy per nucleon compared to the other two isotopes. Therefore, option C is the correct answer. 17 8O because it lies between two stable isotopes.

Oxygen has three stable isotopes: 16 8O, 17 8O, and 18 8O. Binding energy per nucleon is the amount of energy required to disassemble a nucleus into its individual protons and neutrons. Generally, isotopes with a higher binding energy per nucleon are more stable.

17 8O likely has the largest binding energy per nucleon because it lies between two stable isotopes.
Isotopes are atoms of the same element that have different numbers of neutrons in their nucleus. Binding energy per nucleon refers to the amount of energy required to break apart the nucleus of an atom and is a measure of its stability.

The larger the binding energy per nucleon, the more stable the nucleus. The stability of a nucleus depends on the ratio of protons to neutrons in the nucleus. If the ratio is too high or too low, the nucleus becomes unstable and can decay.

Learn more about binding energy here:

brainly.com/question/30073915

#SPJ11

A disk can rotate about its central axis like a merry-go-round. Determine the sign of the angular displacement for the following pairs of initial and final angular positions. The initial angular position is -3 rad and the final angular position is +5 rad.

Answers

The angular displacement of a rotating disk, starting at -3 rad and ending at +5 rad with clockwise rotation, is +8 rad.

If the initial angular position is -3 rad and the final angular position is +5 rad, and the direction of rotation is clockwise as viewed from a clock hanging on a vertical wall, then the angular displacement can be calculated as

Angular displacement = Final angular position - Initial angular position

= (+5 rad) - (-3 rad)

= +8 rad

Since the direction of rotation is clockwise, the angular displacement is positive.

Therefore, the sign of the angular displacement is positive (+) for the given pair of initial and final angular positions.

To know more about angular displacement:

https://brainly.com/question/26483834

#SPJ4

--The given question is incomplete, the complete question is given

" A disk can rotate about its central axis like a merry-go-round. Determine the sign of the angular displacement for the following pairs of initial and final angular positions. The initial angular position is -3 rad and the final angular position is +5 rad. the direction of rotation as you view a clock hanging on vertical wall

A positively charged rod is brought close to an uncharged electroscope. While the rod is close, one's finger touches the far side of the metal ball on the electroscope. The finger is removed and then the rod is taken away. The electroscope is

Answers

When a positively charged rod is brought close to an uncharged electroscope, the electroscope will become polarized. This means that the electrons in the electroscope will move away from the positively charged rod and towards the far end of the electroscope.

This results in a separation of charges within the electroscope, with the far end becoming negatively charged and the near end becoming positively charged.

When one's finger touches the far side of the metal ball on the electroscope, any excess charge on the electroscope is conducted away by the finger. This effectively neutralizes the electroscope, since any excess charge has been transferred to the person's body.

When the finger is removed and the rod is taken away, the electroscope is still neutral. This is because the charge on the rod did not have enough time to transfer to the electroscope before it was neutralized by the finger. Therefore, the electroscope remains in its original uncharged state.

In summary, bringing a positively charged rod close to an uncharged electroscope will cause polarization, touching the electroscope with a finger will neutralize any excess charge, and removing the rod will not leave the electroscope with a net charge.

For more such questions on Electroscope.

https://brainly.com/question/10470196#

#SPJ11

After landing on an unfamiliar planet, a space explorer constructs a simple pendulum of length 54.0 cm. The explorer finds that the pendulum completes 99.0 full swing cycles in a time of 139s. What is the magnitude of the gravitational acceleration on this planet?
g_planet = ___ m/s^2

Answers

The magnitude of the gravitational acceleration on the unfamiliar planet is approximately 2.83 m/s^2.To find the gravitational acceleration on the unfamiliar planet, we can use the formula:

g = (4π²L) / T²

where g is the gravitational acceleration, L is the length of the pendulum, and T is the time for one full swing cycle.

Substituting the given values, we get:

g = (4π² × 0.54 m) / (99.0 × 2 × π)^2 × 139^2 s^2

Simplifying this expression, we get:

g = 2.83 m/s^2

Therefore, the magnitude of the gravitational acceleration on the unfamiliar planet is approximately 2.83 m/s^2.
To calculate the gravitational acceleration (g_planet) on the unfamiliar planet using the simple pendulum, you can follow these steps:

1. Determine the period (T) of the pendulum, which is the time it takes to complete one full swing cycle.

T = total time / number of cycles = 139s / 99.0 cycles = 1.40404s

2. Use the formula for the period of a simple pendulum:

T = 2π * √(L / g_planet)

Where L is the length of the pendulum (0.54 m), and g_planet is the gravitational acceleration we want to find.

3. Rearrange the formula to solve for g_planet:

g_planet = (4π² * L) / T²

4. Plug in the values:

g_planet = (4π² * 0.54 m) / (1.40404s)²

5. Calculate the result:

g_planet ≈ 9.60 m/s²

So, the magnitude of the gravitational acceleration on this unfamiliar planet is approximately 9.60 m/s².

learn more about gravitational acceleration here; brainly.com/question/28188525

#SPJ11

Other Questions
Which answer has the colors in order from the lowest frequency to the highest?Entry field with correct answer Red Blue Green Blue Red Green Blue Green Red Red Green Blue aluminum chloride forms by reaction of 13.43 g of Al with 53.18g of chlorine. what is the % composition of Cl in the compound? To help restore a beach, sand is being added to the beach at a rate of s(t) = 65+ 24 sin (0.3) tons per hour, where t is measured in hours since 5:00 A.M. How many tons of sand are added to the beach over the 3-hour period from 7:00 A.M. to 10:00 AM.? (A) 255.368 (B) 225.271 (C) 85.123 (D) 10.388 what is expected psychosocial development (Erikson: intimacy vs isolation): young adult (20-35 yrs) Which statement is true about TCP reset attacks?A. A TCP reset attack is designed to disrupt the TCP 3-way handshake.B. A TCP reset attack terminates TCP communications between two hosts.C. An malicious attack is always indicated when the RST bit is set to 1 in a TCP packet header.D. In a TCP reset attack, the RST bit in the TCP packet header must be set to 1; settings for other fields in the TCP header are irrelevant. Although Handel was born in Germany, schooled in Italy, he spent a majority of his professional life in ________ 24) The metabolic breakdown of one molecule of glucose generates the greatest amount of ATP energyduring Can yall help me with this please PLEASE HELP!! ASAP PLEASW Construction or creation from a diverse range of available things is called? which cultural dimensions were identified by the dutch researcher geert hofstede? (choose every correct answer.) when an action potential occurs, what happens to the Na / K ions? after this happens, what does the Na/ K ATPpump do? (3 points + 1 for comm.) Let f(t) be some function that satisfies 81 f(t)dt = 1. Evaluate 21 x^2 f(x^3) dx. during the operation of a voltaic cell, the emf decreases over time and q increases. group of answer choices true false the number of registered voters in the voting districts in a county where the districts are drawn fairly. would you be more interested in looking at the mean, median, or mode? state your reasoning. Consider the titration of 25.0 mL of 0.500 M HCl with 0.500 M NaOH. Find the pH in the four Regions.Region 1: 0.00 mL of NaOH addedRegion 2: 12.5 mL of NaOH addedRegion 3: 25.0 mL of NaOH addedRegion 4: 25.1 ml of NaOH added Can pluripotent cells (embryonic stem cells) be used for cloning? ______ refers to a type of pricing that is recommended to be used with a customer of the maintenance department in which he/she is billed a specified amount for a particular job regardless of how many hours are actually labored. Accordingly, the maintenance technician performing the labor is also compensated a specified amount for the same particular job, also regardless of how many hours are actually labored. O Fixed-rate pricing O Flat-rate pricing O Frozen-rate pricing How to list a folder in the databricks dbfs using:1- dbutils.fs API2- %fs magic command London practices the piano 525 minutes in 3 weeks. Assuming she practices the same amount every week, how many minutes would she practice in 4 weeks?